5
$\begingroup$

Inspired by the answers in a past question I asked on techniques for computing integrals using the Feynman parametrization, I became interested in computing integrals over Feynman parameters like so:

  1. $$\int_0^1\int_{0}^{1-u_1} \frac{u_1u_2\sqrt{u_1u_2(1-u_1-u_2)}}{(u_1(1-u_1)\|\textbf{a}\|^2-2u_1u_2\textbf{a}\cdot\textbf{b}+u_2(1-u_2)\|\textbf{b}\|^2)^3} \,du_2du_1$$
  2. $$\int_0^1\int_{0}^{1-u_1} \frac{u_1(u_1-1)\sqrt{u_1u_2(1-u_1-u_2)}}{(u_1(1-u_1)\|\textbf{a}\|^2-2u_1u_2\textbf{a}\cdot\textbf{b}+u_2(1-u_2)\|\textbf{b}\|^2)^3} \,du_2du_1$$
  3. $$\int_0^1\int_{0}^{1-u_1} \frac{u_2(u_2-1)\sqrt{u_1u_2(1-u_1-u_2)}}{(u_1(1-u_1)\|\textbf{a}\|^2-2u_1u_2\textbf{a}\cdot\textbf{b}+u_2(1-u_2)\|\textbf{b}\|^2)^3} \,du_2du_1$$

...where $\textbf{a},\textbf{b}\in\mathbb{R}^3$ are generic vectors.

My question: Does anyone know how to compute the integrals 1. 2. or 3. above or have any useful techniques to share possibly using some of the symmetries present?

Solution Attempt: As these are integrals over a simplex, I have seen related integrals (like the one in the linked question above) performed by making a change of variables like so: $u_1=s(1-t),u_2=(1-s)(1-t), du_2du_1=(1-t)dtds$, so that the integrals are transformed to integrals over a unit square:

1'. $$\int_0^1\int_{0}^{1} \frac{s(1-s)(1-t)\sqrt{ts(1-s)}}{\big(\left(s(1-s)\|\textbf{a}-\textbf{b}\|^2+t\|s\textbf{a}+(1-s)\textbf{b}\|^2 \right)\big)^3} \,dtds$$

2'. $$\int_0^1\int_{0}^{1} \frac{s\big(1-s(1-t)\big)\sqrt{ts(1-s)}}{\big(\left(s(1-s)\|\textbf{a}-\textbf{b}\|^2+t\|s\textbf{a}+(1-s)\textbf{b}\|^2 \right)\big)^3} \,dtds$$

3'. $$\int_0^1\int_{0}^{1} \frac{(1-s)(s(1-t)+t)\sqrt{ts(1-s)}}{\big(\left(s(1-s)\|\textbf{a}-\textbf{b}\|^2+t\|s\textbf{a}+(1-s)\textbf{b}\|^2 \right)\big)^3} \,dtds$$

Mathematica can compute the $t$ integrals in a fairly straightforward way, however it struggles with the $s$ integrals. I tried as well to make a change of variables $s=\frac{u^2}{1+u^2}$ and attempted applying some residue calculations like I've seen applied elsewhere, however I wasn't able to get too far.

Any clues or leads on how Feynman parameters like these are integrated over would be very much appreciated!

(P.S. I can also summarize how I came to these integrals if anyone is interested. It ultimately comes from the Feynman parametrization with multiple denominators where $\alpha_1=\alpha_2=\alpha_3=3/2$)

$\endgroup$

3 Answers 3

1
+100
$\begingroup$

The Feynman parameterization refers to an integral expression of a scattering cross section in terms of three vectors that form a triangle, $\textbf{a},\textbf{b},$ and $\textbf{c}=\textbf{a}-\textbf{b}$. I am not aware of a closed form expression for a general triangle, but an answer in terms of elementary functions exists for an isosceles triangle, $\|\textbf{a}\|=\|\textbf{b}\|\equiv M$, $\textbf{a}\cdot\textbf{b}=M\cos\phi$.

For the isosceles case the first integral in the OP has the form $$I_1(M,\phi)=M^{-6}\int_0^1\!\int_{0}^{1-u_1} \frac{u_1u_2\sqrt{u_1u_2(1-u_1-u_2)}} {\big(u_1(1-u_1)-2u_1u_2\cos\phi+u_2(1-u_2)\big)^3}\,du_2\,du_1.$$ A simple answer results for an equilateral triangle ($\phi=\pi/3$), when $I_1=\tfrac{1}{9}\pi M^{-6}.$ The calculation for general $\phi\in(0,\pi)$ is a bit cumbersome (see below), the result is $$I_1(M,\phi) = \frac{\pi}{M^6} \frac{1+2 \sin \phi/2}{64 \sin ^3(\phi/2) \bigl(1+\sin \phi/2\bigr)^2}.$$

The second integral in the OP follows similarly: $$I_2(M,\phi)=M^{-6}\int_0^1\!\int_{0}^{1-u_1} \frac{u_1(u_1-1)\sqrt{u_1u_2(1-u_1-u_2)}} {\big(u_1(1-u_1)-2u_1u_2\cos\phi+u_2(1-u_2)\big)^3}\,du_2\,du_1. $$ It equals $-\tfrac{2}{9}\pi M^{-6}$ for the equilateral case ($\phi=\pi/3$), while for general $\phi$ it is given by $$I_2(M,\phi)=-\frac{\pi}{M^6}\,\frac{1+4\sin^{3}\phi/2 + 6\sin^{2}\phi/2 + 2\sin\phi/2} {64 \sin ^3(\phi/2) \bigl(1+\sin \phi/2\bigr)^2}.$$

The third integral is equal to $I_2$ in the isosceles case.


Calculation of the integrals (Mathematica was of little help, I asked AI for advise):
Transform variables to $u_1+u_2=s$, $u_1-u_2=st$, \begin{eqnarray} &I_1(M,\phi)=M^{-6}\frac18 \int_0^1 s\sqrt{1-s}\,J(s)\,ds,\;\;J(s)=\int_0^1\frac{(1-t^2)^{3/2}}{(A-Bt^2)^3}\,dt,\\ &A=1-\tfrac12(1+\cos\phi)s,\;\; B=\tfrac12(1-\cos\phi)s. \end{eqnarray} The integral over $t$ is readily evaluated $$ J(s)=\frac{3\pi}{16}\,A^{-5/2}\big(A-B\big)^{-1/2}. $$ Since $A-B=1-s$, the square-root singularity exactly cancels the factor $\sqrt{1-s}$ in the integral over $s$, leaving $$ I_1(M,\phi)=M^{-6}\frac{3\pi}{128}\int_0^1 s\,A^{-5/2}\,ds, $$ which evaluates to the formula for $I_1$ given above.

The integral $I_2$ is evaluated similarly: The coordinate transformation gives \begin{eqnarray} &I_2(M,\phi)=M^{-6}\frac18 \int_0^1 \sqrt{1-s}\bigl[(s-2)J_1(s)+sJ_2(s)\bigr]\,ds,\\ &J_1(s)=\int_0^1\frac{(1-t^2)^{1/2}}{(A-Bt^2)^3}\,dt,\;\; J_2(s)=\int_0^1\frac{t^2(1-t^2)^{1/2}}{(A-Bt^2)^3}\,dt. \end{eqnarray} Evalation of the integral over $t$ results in $$ I_2(M,\phi)=-M^{-6}\frac{\pi}{128}\int_0^1 s\,A^{-5/2}\frac{(4+\cos\phi)s^2-(12+\cos\phi)s+8}{1-s}\,ds, $$ which then produces the formula given above.

If we abandon the isosceles assumption, the same parameter change still allows to carry out the integral over $t$, but then the integral over $s$ contains elliptic functions which prevent further simplification.

$\endgroup$
6
  • 1
    $\begingroup$ Thank you Carlo - thinking through your technique a bit more now, the isosceles case elucidates a lot I think $\endgroup$ Commented Nov 30 at 19:16
  • $\begingroup$ I saw your edit ("If we abandon the isosceles assumption, the same parameter change still allows to carry out the integral over t, but then the integral over s contains elliptic functions which prevent further simplification.") and tried it myself too. Do you mean the integrand over s involves elliptic functions? When I did integral 1) on my side I found the Jacobian of your coordinate change was s/2, and the bounds of integration were $s\in(0,1)$ and $t\in (-1,1)$. The integral over $s\in(0,1)$ integral had a somewhat simplified form, I'll paste it in the next comment: $\endgroup$ Commented Dec 2 at 20:52
  • 1
    $\begingroup$ when I repeat the same parameterization for $\|a\|=x\|b\|$, so non-isosceles triangle for $x\neq 1$, I find a function $J(s)$ that is an elliptic integral, not reducible to an elementary function of $s$ unless $x=1$; then I am unable to carry out the remaining integration over $s$ in closed form; it is actually quite remarkable that such a relatively simple answer follows in the isosceles case. $\endgroup$ Commented Dec 2 at 21:58
  • 1
    $\begingroup$ your recommendation above worked very well, and I was able to get the 1) integral in closed form for $x\neq 1$ and and generic $\phi$. I am rewarding the bounty to you because your method (+Mathematica) resulted in a closed form (below). I'll write up all the steps below yours soon :) $\mathrm{1 integral} = \frac{\pi \sec ^4\left(\frac{\phi}{2}\right) \left(x^2 \left(\sqrt{-2x\cos(\phi)+x^2+1}-x\right)+x \cos(\phi) \left(-2 \sqrt{-2 x \cos (\phi)+x^2+1}+3 x+3\right)+\sqrt{-2 x \cos (\phi)+x^2+1}-1\right)}{32 (\textbf{b}.\textbf{b})^3 x^3 \left(-2 x \cos (\phi)+x^2+1\right)^{3/2}}$ $\endgroup$ Commented Dec 4 at 20:03
  • 1
    $\begingroup$ impressive, I had not thought the general case would work out; thank you for the bounty, I enjoyed working on this problem. $\endgroup$ Commented Dec 4 at 21:19
3
$\begingroup$

$\let\vec=\mathbf$I only discuss integral 1'. Note that there are typos in 1'-3', the denominator should start with $s(1-s) \|\vec a-\vec b\|$ instead of $s(s-1) \|\vec a-\vec b\|$.

Define $\hat{\vec s}=\vec a+\vec b$ and $\hat{\vec d}=\vec a-\vec b$. Rotate the coordinate system such that $\hat{\vec d}$ points in the $z$-direction and $\vec a,\vec b$ lie in the $xz$-plane, and rescale by $\ell=\|\hat{\vec d}\|$ such that $\hat{\vec d} \mapsto \vec d=\{0,0,1\}$ and $\hat{\vec s} \mapsto \vec s = \{ s_x,0,s_z\}$. Then, we evaluate the integral over $t$ and substitiute $s=\sin^2(\sigma/2)$ to get \begin{align}\tag{1a}\label{eq:1a} \frac{I_1}{\pi}&=\frac{1}{2\pi\,\ell^6}\int_{-\pi}^{\pi}\frac{\mathrm d\sigma}{\sin^2\sigma} \left( \frac{A-3}{A^{5/2}}\arctan\sqrt A + \frac{A+3}{A^2(A+1)} \right),\\ \text{with }\quad\tag{1b}\label{eq:1b} A&=\frac{s_x^2+s_z^2-2s_z\cos\sigma+\cos^2\sigma}{\sin^2\sigma}. \end{align} While I found no direct evaluation if this integral (yet), it turned out that $I_1/\pi$ for $s_x=1,\ldots,10$ and $s_z=1,\ldots,10$ could be identified as roots of fourth order polynomials with integer coefficients, using high precision integration and Mathematica's ToRadicals@RootApproximant[I1/Pi,4]. The radicals were simple enough that they could be reverse engineered using FindSequenceFunction[] and some NI (natural intelligence).

The result is surprisingly simple if written with the two roots \begin{align} r_1 &= \sqrt{(s_x^2+s_z^2+1)^2-4s_z^2}, \tag{2a}\label{eq:2a}\\ r_2 &= \frac12(s_x^2+s_z^2-1+r_1), \tag{2b}\label{eq:2b} \end{align} and reads \begin{align}\tag{3}\label{eq:3} \frac{I_1}{\pi}&=\frac{1}{\ell^6} \frac{2+(r_2-2)\sqrt{r_2+1}}{r_1 r_2^2}\,. \end{align} Maybe one can guess an integration scheme from this result.

Finally some Mathematica code to check:

Block[{a = {1,2,3}, b = {4,5,6},M,l,s,d,sx,sz,A,Az,r1,r2}, s=a+b; d=a-b;
 M=RotationMatrix[{d, {0, 0, 1}}]; l = Norm[d];
 a=Simplify[M.a/l];b=Simplify[M.b/l];s=Simplify[a+b];d=Simplify[a-b];
 sx = Simplify@Sqrt[s[[1]]^2 + s[[2]]^2]; sz = s[[3]];
 l^-6 {NIntegrate[(u1 u2 Sqrt[u1 u2 (1-u1-u2)])/
   (u1(1-u1) a.a-2u1 u2 a.b + u2(1-u2) b.b)^3, {u1,0,1}, {u2,0,1-u1}],
   Quiet@NIntegrate[(s(1-s)(1-t) Sqrt[(1-s) s t])/
   (s(1-s)(a-b).(a-b) + t(s a+(1-s)b).(s a+(1-s)b))^3, {s,0,1}, {t,0,1}],
   4 Quiet@NIntegrate[(
      Cos[\[Tau]]^3 Sin[\[Sigma]]^4/Sin[\[Tau]]^4)/(Sin[\[Sigma]]^2/
      Sin[\[Tau]]^2 + s.s - 2s[[3]]Cos[\[Sigma]]+Cos[\[Sigma]]^2)^3,
      {\[Sigma],0, 2\[Pi]}, {\[Tau], 0, \[Pi]/2}],
   A = (sx^2 + sz^2 - 2 sz Cos[\[Sigma]] + Cos[\[Sigma]]^2)/Sin[\[Sigma]]^2; 
   NIntegrate[1/(2 Sin[\[Sigma]]^2) ((A-3)/A^(5/2) ArcTan[Sqrt[A]]
   + (A+3)/(A^2(A+1))), {\[Sigma],-\[Pi],\[Pi]}],
   Az = -((1+2(1+2(sx^2+sz^2))z^2+z^4-4 sz(z+z^3))/(1-z^2)^2); 
   NContourIntegrate[-((2 z)/(I (1-z^2)^2)) ((ArcTan[Sqrt[Az]] (Az-3))/Az^(5/2)
    +(Az+3)/(Az^2 (Az+1))), z \[Element] Circle[]],
   N@\[Pi] (2+(r2-2) Sqrt[r2+1])/(r1 r2^2)//.
   {r1 -> Sqrt[(sx^2+sz^2+1)^2-4 sz^2], r2 -> 1/2(sx^2+sz^2-1+r1)}}]
$\endgroup$
2
  • 1
    $\begingroup$ Thank you! Reviewing this, love the NI :) $\endgroup$ Commented Dec 4 at 20:14
  • 2
    $\begingroup$ deeply impressive; (I made a mistake in an earlier check, it all works out) $\endgroup$ Commented Dec 4 at 21:42
1
$\begingroup$

Just to write up how I got the 1' integral to work using Mathematica and inspired by Carlo's idea:

(*Step 1: t-antideriviative: *)
1/(B.B)^3*
 Integrate[((s*(1 - s)*(1 - t)*
      Sqrt[t*s*(1 - s)])/(s*(1 - s)*(x^2 - 2*x*Cos[T] + 1) + 
       t*(s^2*x^2 + (1 - s)^2 + 2*s*(1 - s)*x*Cos[T]))^3), t]

(*Step 2 apply limit t->1 then t->0*)
Limit[1/(B.B)^3 (-1 + 
     s) s (-((Sqrt[-(-1 + s) s t]
           ArcTan[(
           Sqrt[t] Sqrt[-1 + 2 s - s^2 - s^2 x^2 - 2 s x Cos[T] + 
             2 s^2 x Cos[T]])/(
           Sqrt[1 - s] Sqrt[s] Sqrt[-1 - x^2 + 2 x Cos[T]])] (-1 + 
            5 s - 4 s^2 + 3 s x^2 - 4 s^2 x^2 - 8 s x Cos[T] + 
            8 s^2 x Cos[T]))/(4 Sqrt[1 - s] (-1 + s) s^(3/2) Sqrt[
          t] (-1 - x^2 + 2 x Cos[T])^(
          3/2) (-1 + 2 s - s^2 - s^2 x^2 - 2 s x Cos[T] + 
            2 s^2 x Cos[T])^(5/2))) + (Sqrt[-(-1 + s) s t] (1 - s + 
          s x^2))/(2 (-1 + 2 s - s^2 - s^2 x^2 - 2 s x Cos[T] + 
          2 s^2 x Cos[T])^2 (-s + s^2 - t + 2 s t - s^2 t - s x^2 + 
          s^2 x^2 - s^2 t x^2 + 2 s x Cos[T] - 2 s^2 x Cos[T] - 
          2 s t x Cos[T] + 
          2 s^2 t x Cos[T])^2) + (Sqrt[-(-1 + s) s t] (1 + 3 s - 
          4 s^2 + 5 s x^2 - 4 s^2 x^2 - 8 s x Cos[T] + 
          8 s^2 x Cos[T]))/(4 (-1 + s) s (-1 - x^2 + 
          2 x Cos[T]) (-1 + 2 s - s^2 - s^2 x^2 - 2 s x Cos[T] + 
          2 s^2 x Cos[T])^2 (-s + s^2 - t + 2 s t - s^2 t - s x^2 + 
          s^2 x^2 - s^2 t x^2 + 2 s x Cos[T] - 2 s^2 x Cos[T] - 
          2 s t x Cos[T] + 2 s^2 t x Cos[T]))), t -> 1] - 
 Limit[1/(B.B)^3 (-1 + 
     s) s (-((Sqrt[-(-1 + s) s t]
           ArcTan[(
           Sqrt[t] Sqrt[-1 + 2 s - s^2 - s^2 x^2 - 2 s x Cos[T] + 
             2 s^2 x Cos[T]])/(
           Sqrt[1 - s] Sqrt[s] Sqrt[-1 - x^2 + 2 x Cos[T]])] (-1 + 
            5 s - 4 s^2 + 3 s x^2 - 4 s^2 x^2 - 8 s x Cos[T] + 
            8 s^2 x Cos[T]))/(4 Sqrt[1 - s] (-1 + s) s^(3/2) Sqrt[
          t] (-1 - x^2 + 2 x Cos[T])^(
          3/2) (-1 + 2 s - s^2 - s^2 x^2 - 2 s x Cos[T] + 
            2 s^2 x Cos[T])^(5/2))) + (Sqrt[-(-1 + s) s t] (1 - s + 
          s x^2))/(2 (-1 + 2 s - s^2 - s^2 x^2 - 2 s x Cos[T] + 
          2 s^2 x Cos[T])^2 (-s + s^2 - t + 2 s t - s^2 t - s x^2 + 
          s^2 x^2 - s^2 t x^2 + 2 s x Cos[T] - 2 s^2 x Cos[T] - 
          2 s t x Cos[T] + 
          2 s^2 t x Cos[T])^2) + (Sqrt[-(-1 + s) s t] (1 + 3 s - 
          4 s^2 + 5 s x^2 - 4 s^2 x^2 - 8 s x Cos[T] + 
          8 s^2 x Cos[T]))/(4 (-1 + s) s (-1 - x^2 + 
          2 x Cos[T]) (-1 + 2 s - s^2 - s^2 x^2 - 2 s x Cos[T] + 
          2 s^2 x Cos[T])^2 (-s + s^2 - t + 2 s t - s^2 t - s x^2 + 
          s^2 x^2 - s^2 t x^2 + 2 s x Cos[T] - 2 s^2 x Cos[T] - 
          2 s t x Cos[T] + 2 s^2 t x Cos[T]))), t -> 0]

(*Step 3: take the result from above and integrate over s *)

Integrate[
 1/(B.B)^3 (-1 + s) s ((Sqrt[(1 - s) s] (1 - s + s x^2))/(
    2 (-1 + s - s x^2)^2 (-1 + 2 s - s^2 - s^2 x^2 - 2 s x Cos[T] + 
       2 s^2 x Cos[T])^2) - (Sqrt[(1 - s) s]
        ArcTan[Sqrt[-1 + 2 s - s^2 - s^2 x^2 - 2 s x Cos[T] + 
         2 s^2 x Cos[T]]/(
        Sqrt[1 - s] Sqrt[s] Sqrt[-1 - x^2 + 2 x Cos[T]])] (-1 + 5 s - 
         4 s^2 + 3 s x^2 - 4 s^2 x^2 - 8 s x Cos[T] + 
         8 s^2 x Cos[T]))/(4 Sqrt[1 - s] (-1 + s) s^(
       3/2) (-1 - x^2 + 2 x Cos[T])^(
       3/2) (-1 + 2 s - s^2 - s^2 x^2 - 2 s x Cos[T] + 
         2 s^2 x Cos[T])^(
       5/2)) + (Sqrt[(1 - s) s] (1 + 3 s - 4 s^2 + 5 s x^2 - 
         4 s^2 x^2 - 8 s x Cos[T] + 8 s^2 x Cos[T]))/(4 (-1 + 
         s) s (-1 + s - s x^2) (-1 - x^2 + 2 x Cos[T]) (-1 + 2 s - 
         s^2 - s^2 x^2 - 2 s x Cos[T] + 2 s^2 x Cos[T])^2)), s, 
 Assumptions -> (Element[x, Reals] && (x > 0) && 
    Element[T, Reals] && (T >= 0) && (T <= 2*Pi))]

(*Step 4 - FTC...but weird -(F(1) + F(0)) where F(s) is the antiderivative from above...must arise from a choice of branch cut in Integrate*)

-(Limit[(ArcTan[
         Sqrt[-1 + 2 s - s^2 (1 + x^2) + 2 (-1 + s) s x Cos[T]]/(
         Sqrt[-(-1 + s) s]
           Sqrt[-1 - x^2 + 
           2 x Cos[
             T]])] (-(-8 - 21 x^2 - 24 s^2 (1 + 8 x^2 + 4 x^4) + 
              3 s (8 + 39 x^2 + 7 x^4) + 
              8 s^3 (1 + 12 x^2 + 12 x^4 + x^6)) Cos[T] + 
          x (2 (-1 + 2 s) (6 + x^2 - 4 s (3 + 5 x^2) + 
                s^2 (6 + 20 x^2 + 6 x^4)) Cos[2 T] - 
             3 (1 - 8 s + 8 s^2) x (-1 + s + s x^2) Cos[3 T] + 
             2 (-1 + 2 s) (6 + 3 x^2 - 6 s (2 + 3 x^2) + 
                6 s^2 (1 + 3 x^2 + x^4) + 
                2 (-1 + s) s x^2 Cos[4 T]))) Csc[
         T]^4)/(16 x^3 (-1 - x^2 + 2 x Cos[T])^(
        3/2) (-1 + 2 s - s^2 (1 + x^2) + 2 (-1 + s) s x Cos[T])^(
        3/2) (B.B)^3) + (Sqrt[-(-1 + s) s]
         Csc[T]^4 ((
          2 ArcTanh[x/Sqrt[(-1 + s)/
            s]] (1 + x^2 - 2 x Cos[T]) (3 + Cos[2 T]))/
          Sqrt[-1 + s] + (
          8 Cos[T] (-1 - x^2 + 2 x Cos[T]) Log[
            Sqrt[-1 + s] + Sqrt[s]])/Sqrt[-1 + s] - (
          4 Sqrt[s]
            x (-1 + s + s x^2 + (x - 2 s x) Cos[T]) Sin[T]^2)/(
          1 - 2 s + s^2 (1 + x^2) - 2 (-1 + s) s x Cos[T])))/(16 Sqrt[
        s] x^3 (-1 - x^2 + 2 x Cos[T]) (B.B)^3), s -> 1, 
    Assumptions -> (Element[x, Reals] && (x > 0) && 
       Element[T, Reals] && (T >= 0) && (T <= 2*Pi))] + 
   Limit[(ArcTan[
         Sqrt[-1 + 2 s - s^2 (1 + x^2) + 2 (-1 + s) s x Cos[T]]/(
         Sqrt[-(-1 + s) s]
           Sqrt[-1 - x^2 + 
           2 x Cos[
             T]])] (-(-8 - 21 x^2 - 24 s^2 (1 + 8 x^2 + 4 x^4) + 
              3 s (8 + 39 x^2 + 7 x^4) + 
              8 s^3 (1 + 12 x^2 + 12 x^4 + x^6)) Cos[T] + 
          x (2 (-1 + 2 s) (6 + x^2 - 4 s (3 + 5 x^2) + 
                s^2 (6 + 20 x^2 + 6 x^4)) Cos[2 T] - 
             3 (1 - 8 s + 8 s^2) x (-1 + s + s x^2) Cos[3 T] + 
             2 (-1 + 2 s) (6 + 3 x^2 - 6 s (2 + 3 x^2) + 
                6 s^2 (1 + 3 x^2 + x^4) + 
                2 (-1 + s) s x^2 Cos[4 T]))) Csc[
         T]^4)/(16 x^3 (-1 - x^2 + 2 x Cos[T])^(
        3/2) (-1 + 2 s - s^2 (1 + x^2) + 2 (-1 + s) s x Cos[T])^(
        3/2) (B.B)^3) + (Sqrt[-(-1 + s) s]
         Csc[T]^4 ((
          2 ArcTanh[x/Sqrt[(-1 + s)/
            s]] (1 + x^2 - 2 x Cos[T]) (3 + Cos[2 T]))/
          Sqrt[-1 + s] + (
          8 Cos[T] (-1 - x^2 + 2 x Cos[T]) Log[
            Sqrt[-1 + s] + Sqrt[s]])/Sqrt[-1 + s] - (
          4 Sqrt[s]
            x (-1 + s + s x^2 + (x - 2 s x) Cos[T]) Sin[T]^2)/(
          1 - 2 s + s^2 (1 + x^2) - 2 (-1 + s) s x Cos[T])))/(16 Sqrt[
        s] x^3 (-1 - x^2 + 2 x Cos[T]) (B.B)^3), s -> 0, 
    Assumptions -> (Element[x, Reals] && (x > 0) && 
       Element[T, Reals] && (T >= 0) && (T <= 2*Pi))])

Of note in the Mathematica code above: the final result is obtained by summing the limits of the s-antiderivatve obtained in step 3 as s->1 and s->0. I am not sure why this works, I'm assuming it comes from some choice of branch cut in Mathematica's symbolic integration scheme.

The final result after some simplification is then: $\frac{\pi \sec ^4\left(\frac{T}{2}\right) \left(3 x^2 \cos (T)+\left(-2 x \cos (T)+x^2+1\right)^{3/2}+3 x \cos (T)-x^3-1\right)}{32 (B.B)^3 x^3 \left(-2 x \cos (T)+x^2+1\right)^{3/2}}$

where $x=\|\vec a\|/\|\vec b\|$ and $T=ArcCos[\vec a \cdot \vec b / \|\vec a\|/\|\vec b\|]$

(P.S. Please forgive the sloppy code above, it is the only way I can depict how I got the integral working...weirdly, suppressing output with ";", or by assigning symbolic results to variables to make the code more readable, gives superficially different results, and wrong final answers when compared to:

NIntegrate[
 NIntegrate[
  u2*u1*Sqrt[
     u1*u2*(1 - u1 - u2)]/(u1*(1 - u1)*A.A - 2*u1*u2*A.B + 
       u2*(1 - u2)*B.B)^3, {u2, 0, 1 - u1}], {u1, 0, 1}]

)

$\endgroup$

You must log in to answer this question.

Start asking to get answers

Find the answer to your question by asking.

Ask question

Explore related questions

See similar questions with these tags.